Difference between revisions of "2014 AIME I Problems"

(Problem 3)
(Please do not post problems from the AIME until the AoPS AIME forum reopens.)
Line 7: Line 7:
  
 
==Problem 2==  
 
==Problem 2==  
An urn contains <math>4</math> green balls and <math>6</math> blue balls. A second urn contains <math>16</math> green balls and <math>N</math> blue balls. A single ball is drawn at random from each urn. The probability that both balls are of the same color is 0.58. Find <math>N</math>.
+
 
  
  
Line 13: Line 13:
  
 
==Problem 3==
 
==Problem 3==
Find the number of rational numbers <math>r,</math> <math>0<r<1,</math> such that when <math>r</math> is written as a fraction in lowest terms, the numerator and the denominator have a sum of 1000.
+
 
  
 
[[2014 AIME I Problems/Problem 3|Solution]]
 
[[2014 AIME I Problems/Problem 3|Solution]]

Revision as of 10:46, 14 March 2014

2014 AIME I (Answer Key)
Printable version | AoPS Contest CollectionsPDF

Instructions

  1. This is a 15-question, 3-hour examination. All answers are integers ranging from $000$ to $999$, inclusive. Your score will be the number of correct answers; i.e., there is neither partial credit nor a penalty for wrong answers.
  2. No aids other than scratch paper, graph paper, ruler, compass, and protractor are permitted. In particular, calculators and computers are not permitted.
1 2 3 4 5 6 7 8 9 10 11 12 13 14 15

Problem 1

Solution

Problem 2

Solution

Problem 3

Solution

Problem 4

Solution

Problem 5

Solution

Problem 6

Solution

Problem 7

Solution

Problem 8

Solution

Problem 9

Solution

Problem 10

Solution

Problem 11

Solution

Problem 12

Solution

Problem 13

Solution

Problem 14

Solution

Problem 15

Solution

The problems on this page are copyrighted by the Mathematical Association of America's American Mathematics Competitions. AMC logo.png